[Math] Double integral where limits are the first quadrant

definite integralsintegrationmultivariable-calculus

Evaluate the integral $$\iint\limits_D \frac{1}{(x+y+1)^3} \, dA$$ where $D$ is the first quadrant.

In this case, what would the limits of integration be? I'm having trouble moving to polar coordinates. Obviously the unit circle for the first quadrant is the area from $0$ to $\pi/2$, but I'm not sure how to break this up for $x$ and $y$.

Best Answer

Hint: Calculate $$\int_{x=0}^\infty\left(\int_{y=0}^\infty \frac{1}{(x+y+1)^3}\,dy\right)\,dx.$$ The inner integral is straightforward, just a power. Same for the second.

Maybe for the first you can let $u=x+y+1$, but it should not be necessary.